Will you really find roots?

Algebra Level 5

Find the sum of the roots (real or complex, including multiplicities) of the equation x 2001 + ( 1 2 x ) 2001 = 0 x^{2001} + \bigg( \frac{1}{2} - x \bigg)^{2001} = 0


The answer is 500.

This section requires Javascript.
You are seeing this because something didn't load right. We suggest you, (a) try refreshing the page, (b) enabling javascript if it is disabled on your browser and, finally, (c) loading the non-javascript version of this page . We're sorry about the hassle.

2 solutions

Akshay Yadav
Mar 4, 2016

The question is based on Binomial theorem ,

With help of it we will simplify the equation given to us.

x 2001 ( x 1 2 ) 2001 = 0 x^{2001}-(x-\frac{1}{2})^{2001}=0

x 2001 x 2001 + 2001 2 x 2000 500250 x 1999 + . . . . = 0 x^{2001}-x^{2001}+\frac{2001}{2}x^{2000}-500250x^{1999} + ....=0

2001 2 x 2000 500250 x 1999 + . . . . = 0 \frac{2001}{2}x^{2000}-500250x^{1999} + ....=0

x 2000 500 x 1999 + . . . . = 0 x^{2000}-500x^{1999} + ....=0

Hence, by Vieta's formula sum of roots is ( 500 ) 1 = 500 \frac{-(-500)}{1}= \boxed{500}

EDIT: Fixed. Thanks for the help!

Anthony Ritz - 5 years, 3 months ago

Log in to reply

Good solution! I cannot do anything to help you but surely a staff member can. @Calvin Lin @Andrew Ellinor Please help!

Akshay Yadav - 5 years, 3 months ago

Sorry that happened to you Anthony! We'll try to fix it, but in the meantime feel free posting solutions to other problems. Take note that you can edit your solutions instead of deleting them. Happy solving! :)

Andrew Ellinor - 5 years, 3 months ago
Anthony Ritz
Mar 4, 2016

I did this a bit differently, using symmetry considerations:

Let x = 1 4 + y x=\frac{1}{4}+y . The polynomial becomes ( 1 4 + y ) 2001 + ( 1 4 y ) 2001 (\frac{1}{4}+y)^{2001}+(\frac{1}{4}-y)^{2001} , and it is fairly easy to see that its roots, whether real or complex, must be symmetrical around 1 4 \frac{1}{4} . Each symmetrical pair of roots adds up to 1 2 \frac{1}{2} , and there are 2 , 000 2,000 roots in the complex plane since the polynomial has degree 2 , 000 2,000 (note that the degree- 2001 2001 terms cancel). So the answer must be 1 4 2 , 000 = 1 2 1 , 000 = 500 \frac{1}{4}*2,000=\frac{1}{2}*1,000=\boxed{500} .

More generally, for any real number m m and for any odd integer n n , the sum of the complex roots of x n + ( m x ) n x^{n}+(m-x)^{n} ought to be ( n 1 ) m 2 \frac{(n-1)*m}{2} , and for any even integer n n it ought to be n m 2 \frac{n*m}{2} .

I have undeleted your solution, and updated it with your comment.

Thanks for sharing this symmetry argument.

Calvin Lin Staff - 5 years, 3 months ago

0 pending reports

×

Problem Loading...

Note Loading...

Set Loading...